Difference between revisions of "1991 AIME Problems/Problem 13"

(Solution)
(22 intermediate revisions by 7 users not shown)
Line 1: Line 1:
 
== Problem ==
 
== Problem ==
A drawer contains a mixture of red socks and blue socks, at most 1991 in all. It so happens that, when two socks are selected randomly without replacement, there is a probability of exactly <math>\displaystyle \frac{1}{2}</math> that both are red or both are blue. What is the largest possible number of red socks in the drawer that is consistent with this data?  
+
A drawer contains a mixture of red socks and blue socks, at most <math>1991</math> in all. It so happens that, when two socks are selected randomly without replacement, there is a probability of exactly <math>\frac{1}{2}</math> that both are red or both are blue. What is the largest possible number of red socks in the drawer that is consistent with this data?  
  
 
== Solution ==
 
== Solution ==
Let <math>r</math> and <math>b</math> denote the number of red and blue socks, respectively. Also, let <math>t=r+b</math>. The probability <math>P</math> that when two socks are drawn without replacement, both are red or both are blue is given by
+
=== Solution 1 ===
 +
Let <math>r</math> and <math>b</math> denote the number of red and blue socks, respectively. Also, let <math>t=r+b</math>. The probability <math>P</math> that when two socks are drawn randomly, without replacement, both are red or both are blue is given by
  
<math>
+
<cmath>
P=\frac{r(r-1)}{(r+b)(r+b-1)}+\frac{b(b-1)}{(r+b)(r+b-1)}=\frac{r(r-1)+(t-r)(t-r-1)}{t(t-1)}=\frac{1}{2}\,.
+
\frac{r(r-1)}{(r+b)(r+b-1)}+\frac{b(b-1)}{(r+b)(r+b-1)}=\frac{r(r-1)+(t-r)(t-r-1)}{t(t-1)}=\frac{1}{2}.
</math>
+
</cmath>
  
 
Solving the resulting quadratic equation <math>r^{2}-rt+t(t-1)/4=0</math>, for <math>r</math> in terms of <math>t</math>, one obtains that  
 
Solving the resulting quadratic equation <math>r^{2}-rt+t(t-1)/4=0</math>, for <math>r</math> in terms of <math>t</math>, one obtains that  
  
<math>
+
<cmath>
 
r=\frac{t\pm\sqrt{t}}{2}\, .
 
r=\frac{t\pm\sqrt{t}}{2}\, .
</math>
+
</cmath>
  
Now, since <math>r</math> and <math>t</math> are positive integers, it must be the case that <math>t=n^{2}</math>, with <math>n\in\mathbb{N}</math>. Hence, <math>r=n(n\pm1)/2</math> would correspond to the general solution. For the given case, <math>t\leq 1991</math> and so one easily finds that <math>n=44</math>, <math>t=1936</math> and, thus, <math>r=990</math> are the largest possible integers satisfying the problem conditions.  
+
Now, since <math>r</math> and <math>t</math> are positive integers, it must be the case that <math>t=n^{2}</math>, with <math>n\in\mathbb{N}</math>. Hence, <math>r=n(n\pm 1)/2</math> would correspond to the general solution. For the present case <math>t\leq 1991</math>, and so one easily finds that <math>n=44</math> is the largest possible integer satisfying the problem conditions.  
  
In summary, the solution is that the maximum number of red socks is <math>r=990</math>.
+
In summary, the solution is that the maximum number of red socks is <math>r=\boxed{990}</math>.
 +
=== Solution 2 ===
 +
Let <math>r</math> and <math>b</math> denote the number of red and blue socks such that <math>r+b\le1991</math>. Then by complementary counting, the number of ways to get a red and a blue sock must be equal to <math>1-\frac12=\frac12=\frac{2rb}{(r+b)(r+b-1)}\implies4rb=(r+b)(r+b-1)</math>
 +
<math>=(r+b)^2-(r+b)\implies r^2+2rb+b^2-r-b=4rb\implies r^2-2rb+b^2</math>
 +
<math>=(r-b)^2=r+b</math>, so <math>r+b</math> must be a perfect square <math>k^2</math>. Clearly, <math>r=\frac{k^2+k}2</math>, so the larger <math>k</math>, the larger <math>r</math>: <math>k^2=44^2</math> is the largest perfect square below <math>1991</math>, and our answer is <math>\frac{44^2+44}2=\frac12\cdot44(44+1)=22\cdot45=11\cdot90=\boxed{990}</math>.
 +
 
 +
 
 +
===Solution 3===
 +
 
 +
Let <math>r</math> and <math>b</math> denote the number of red and blue socks, respectively. In addition, let <math>t = r + b</math>, the total number of socks in the drawer.
 +
 
 +
From the problem, it is clear that <math>\frac{r(r-1)}{t(t-1)} + \frac{b(b-a)}{t(t-1)} = \frac{1}{2}</math>
 +
 
 +
Expanding, we get <math>\frac{r^2 + b^2 - r - b}{t^2 - t} = \frac{1}{2}</math>
 +
 
 +
Substituting <math>t</math> for <math>r + b</math> and cross multiplying, we get <math>2r^2 + 2b^2 - 2r - 2b = r^2 + 2br + b^2 - r - b</math>
 +
 
 +
Combining terms, we get <math>b^2 - 2br + r^2 - b - r = 0</math>
 +
 
 +
To make this expression factorable, we add <math>2r</math> to both sides, resulting in <math>(b - r)^2 - 1(b-r) = (b - r - 1)(b - r) = 2r</math>
 +
 
 +
From this equation, we can test values for the expression <math>(b - r - 1)(b - r)</math>, which is the multiplication of two consecutive integers, until we find the highest value of <math>b</math> or <math>r</math> such that <math>b + r \leq 1991</math>.
 +
 
 +
By testing <math>(b - r - 1) = 43</math> and <math>(b - r) = 44</math>, we get that <math>r = 43(22) = 946</math> and <math>b = 990</math>. Testing values one integer higher, we get that <math>r = 990</math> and <math>b = 1035</math>. Since <math>990 + 1035 = 2025</math> is greater than <math>1991</math>, we conclude that <math>(946, 990)</math> is our answer.
 +
 
 +
Since it doesn't matter whether the number of blue or red socks is <math>990</math>, we take the higher value for <math>r</math>, thus the maximum number of red socks is <math>r=\boxed{990}</math>.
 +
 
 +
=== Solution 4 ===
 +
As above, let <math>r</math>, <math>b</math>, and <math>t</math> denote the number of red socks, the number of blue socks, and the total number of socks, respectively. We see that <math>\frac{r(r-1)}{t(t-1)}+\frac{b(b-1)}{t(t-1)}=\frac{1}{2}</math>, so <math>r^2+b^2-r-b=\frac{t(t-1)}{2}=r^2+b^2-t=\frac{t^2}{2}-\frac{t}{2}</math>.
 +
 
 +
Seeing that we can rewrite <math>r^2+b^2</math> as <math>(r+b)^2-2rb</math>, and remembering that <math>r+b=t</math>, we have <math>\frac{t^2}{2}-\frac{t}{2}=t^2-2rb-t</math>, so <math>2rb=\frac{t^2}{2}-\frac{t}{2}</math>, which equals <math>r^2+b^2-t</math>.
 +
 
 +
We now have <math>r^2+b^2-t=2rb</math>, so <math>r^2-2rb+b^2=t</math> and <math>r-b=\pm\sqrt{t}</math>. Adding this to <math>r+b=t</math>, we have <math>2r=t\pm\sqrt{t}</math>. To maximize <math>r</math>, we must use the positive square root and maximize <math>t</math>. The largest possible value of <math>t</math> is the largest perfect square less than 1991, which is <math>1936=44^2</math>. Therefore, <math>r=\frac{t+\sqrt{t}}{2}=\frac{1936+44}{2}=\boxed{990}</math>.
 +
 
 +
Solution by Zeroman
 +
 
 +
===Solution 5===
 +
Let <math>r</math> be the number of socks that are red, and <math>t</math> be the total number of socks. We get:
 +
 
 +
<math>2(r(r-1)+(t-r)(t-r-1))=t(t-1)</math>
 +
Expanding the left hand side and the right hand side, we get:
 +
<math>4r^2-4rt+2t^2-2t = t^2-t</math>
 +
 
 +
And, moving terms, we will get that:
 +
<math>4r^2-4rt+t^2 = t</math>
 +
 
 +
We notice that the left side is a perfect square.
 +
<math>(2r-t)^2 = t</math>
 +
 
 +
Thus <math>t</math> is a perfect square. And, the higher <math>t</math> is, the higher <math>r</math> will be. So, we should set <math>t = 44^2 = 1936</math>
 +
 
 +
And, we see, <math>2r-1936 = \pm44</math> We will use the positive root, to get that <math>2r-1936 = 44</math>, <math>2r = 1980</math>, and <math>r = \boxed{990}</math>.
 +
 
 +
- AlexLikeMath
  
 
== See also ==
 
== See also ==
{{AIME box|year=1991|num-b=12|num-a=14}}</math>
+
{{AIME box|year=1991|num-b=12|num-a=14}}
 +
 
 +
[[Category:Intermediate Combinatorics Problems]]
 +
{{MAA Notice}}

Revision as of 19:29, 27 June 2019

Problem

A drawer contains a mixture of red socks and blue socks, at most $1991$ in all. It so happens that, when two socks are selected randomly without replacement, there is a probability of exactly $\frac{1}{2}$ that both are red or both are blue. What is the largest possible number of red socks in the drawer that is consistent with this data?

Solution

Solution 1

Let $r$ and $b$ denote the number of red and blue socks, respectively. Also, let $t=r+b$. The probability $P$ that when two socks are drawn randomly, without replacement, both are red or both are blue is given by

\[\frac{r(r-1)}{(r+b)(r+b-1)}+\frac{b(b-1)}{(r+b)(r+b-1)}=\frac{r(r-1)+(t-r)(t-r-1)}{t(t-1)}=\frac{1}{2}.\]

Solving the resulting quadratic equation $r^{2}-rt+t(t-1)/4=0$, for $r$ in terms of $t$, one obtains that

\[r=\frac{t\pm\sqrt{t}}{2}\, .\]

Now, since $r$ and $t$ are positive integers, it must be the case that $t=n^{2}$, with $n\in\mathbb{N}$. Hence, $r=n(n\pm 1)/2$ would correspond to the general solution. For the present case $t\leq 1991$, and so one easily finds that $n=44$ is the largest possible integer satisfying the problem conditions.

In summary, the solution is that the maximum number of red socks is $r=\boxed{990}$.

Solution 2

Let $r$ and $b$ denote the number of red and blue socks such that $r+b\le1991$. Then by complementary counting, the number of ways to get a red and a blue sock must be equal to $1-\frac12=\frac12=\frac{2rb}{(r+b)(r+b-1)}\implies4rb=(r+b)(r+b-1)$ $=(r+b)^2-(r+b)\implies r^2+2rb+b^2-r-b=4rb\implies r^2-2rb+b^2$ $=(r-b)^2=r+b$, so $r+b$ must be a perfect square $k^2$. Clearly, $r=\frac{k^2+k}2$, so the larger $k$, the larger $r$: $k^2=44^2$ is the largest perfect square below $1991$, and our answer is $\frac{44^2+44}2=\frac12\cdot44(44+1)=22\cdot45=11\cdot90=\boxed{990}$.


Solution 3

Let $r$ and $b$ denote the number of red and blue socks, respectively. In addition, let $t = r + b$, the total number of socks in the drawer.

From the problem, it is clear that $\frac{r(r-1)}{t(t-1)} + \frac{b(b-a)}{t(t-1)} = \frac{1}{2}$

Expanding, we get $\frac{r^2 + b^2 - r - b}{t^2 - t} = \frac{1}{2}$

Substituting $t$ for $r + b$ and cross multiplying, we get $2r^2 + 2b^2 - 2r - 2b = r^2 + 2br + b^2 - r - b$

Combining terms, we get $b^2 - 2br + r^2 - b - r = 0$

To make this expression factorable, we add $2r$ to both sides, resulting in $(b - r)^2 - 1(b-r) = (b - r - 1)(b - r) = 2r$

From this equation, we can test values for the expression $(b - r - 1)(b - r)$, which is the multiplication of two consecutive integers, until we find the highest value of $b$ or $r$ such that $b + r \leq 1991$.

By testing $(b - r - 1) = 43$ and $(b - r) = 44$, we get that $r = 43(22) = 946$ and $b = 990$. Testing values one integer higher, we get that $r = 990$ and $b = 1035$. Since $990 + 1035 = 2025$ is greater than $1991$, we conclude that $(946, 990)$ is our answer.

Since it doesn't matter whether the number of blue or red socks is $990$, we take the higher value for $r$, thus the maximum number of red socks is $r=\boxed{990}$.

Solution 4

As above, let $r$, $b$, and $t$ denote the number of red socks, the number of blue socks, and the total number of socks, respectively. We see that $\frac{r(r-1)}{t(t-1)}+\frac{b(b-1)}{t(t-1)}=\frac{1}{2}$, so $r^2+b^2-r-b=\frac{t(t-1)}{2}=r^2+b^2-t=\frac{t^2}{2}-\frac{t}{2}$.

Seeing that we can rewrite $r^2+b^2$ as $(r+b)^2-2rb$, and remembering that $r+b=t$, we have $\frac{t^2}{2}-\frac{t}{2}=t^2-2rb-t$, so $2rb=\frac{t^2}{2}-\frac{t}{2}$, which equals $r^2+b^2-t$.

We now have $r^2+b^2-t=2rb$, so $r^2-2rb+b^2=t$ and $r-b=\pm\sqrt{t}$. Adding this to $r+b=t$, we have $2r=t\pm\sqrt{t}$. To maximize $r$, we must use the positive square root and maximize $t$. The largest possible value of $t$ is the largest perfect square less than 1991, which is $1936=44^2$. Therefore, $r=\frac{t+\sqrt{t}}{2}=\frac{1936+44}{2}=\boxed{990}$.

Solution by Zeroman

Solution 5

Let $r$ be the number of socks that are red, and $t$ be the total number of socks. We get:

$2(r(r-1)+(t-r)(t-r-1))=t(t-1)$ Expanding the left hand side and the right hand side, we get: $4r^2-4rt+2t^2-2t = t^2-t$

And, moving terms, we will get that: $4r^2-4rt+t^2 = t$

We notice that the left side is a perfect square. $(2r-t)^2 = t$

Thus $t$ is a perfect square. And, the higher $t$ is, the higher $r$ will be. So, we should set $t = 44^2 = 1936$

And, we see, $2r-1936 = \pm44$ We will use the positive root, to get that $2r-1936 = 44$, $2r = 1980$, and $r = \boxed{990}$.

- AlexLikeMath

See also

1991 AIME (ProblemsAnswer KeyResources)
Preceded by
Problem 12
Followed by
Problem 14
1 2 3 4 5 6 7 8 9 10 11 12 13 14 15
All AIME Problems and Solutions

The problems on this page are copyrighted by the Mathematical Association of America's American Mathematics Competitions. AMC logo.png